LSAT and Law School Admissions Forum

Get expert LSAT preparation and law school admissions advice from PowerScore Test Preparation.

User avatar
 Dave Killoran
PowerScore Staff
  • PowerScore Staff
  • Posts: 5853
  • Joined: Mar 25, 2011
|
#88072
Complete Question Explanation
(The complete setup for this game can be found here: lsat/viewtopic.php?f=166&t=8547)

The correct answer choice is (C).

This question asks you to find the additional piece of ranking information that will force all six crews into a single sequence. When this type of question appears in a sequencing game, the usual solution either takes a “front” variable and pushes it towards the back, or takes a “back” variable and pushes it toward the front. These answers are more likely to be correct because they create the most problems for other variables. Answers that keep “front” variables near the front, “middle” variables near the middle, or “back” variables near the back are often wrong because they leave many possibilities available, and are not good answer choices to start with. In this problem, answer choices (A), (D), and (E) match one of the above scenarios. Thus, if you do not have a prephrased answer in mind, you should start your attack on this problem with either (B) or (C).

Answer choice (B) takes G, which appears to be a front variable, and ranks it fifth. The problem with this answer is that G has a great deal of latitude in this game, and although it appears to be a front variable, G must only rank ahead of T, and since ranking G fifth forces T to rank sixth, the remaining four variables have several possible arrangements.

Answer choice (C) takes H, a back variable, and ranks it third. With H ranked third, R must rank first and S must rank second. And, because F :longline: G :longline: T, F must rank fourth, G fifth, and T sixth. Thus, when H ranks third, there is only one solution to the game and answer choice (C) is correct.
 srcline@noctrl.edu
  • Posts: 243
  • Joined: Oct 16, 2015
|
#22113
Hello

For Q 16 I am in between answer choices C D. I think I am having an issue because Im trying to solve for absolutes?
Here are my hypotheticals:

(C): R S H F G T

(D): F G R S T H

Thankyou
Sarah
 Jon Denning
PowerScore Staff
  • PowerScore Staff
  • Posts: 904
  • Joined: Apr 11, 2011
|
#22114
Hey Sarah,

First, you may want to refer to my previous post regarding setup: lsat/viewtopic.php?f=166&t=8547&p=22066#p22066

16. If (D) was correct you'd have R third. That gives the following:

..... F G R (T, S > H) [notice how your hypothetical above isn't the only possibility here]

That's not fixed since the final three can move, so it doesn't work.

With (C), you have H third:

..... R S H F G T [and that's a fixed order]

I hope that helps!

Get the most out of your LSAT Prep Plus subscription.

Analyze and track your performance with our Testing and Analytics Package.